Using rank-nullity theorem to show alternating sum of dimensions = 0

Click For Summary
The discussion focuses on using the rank-nullity theorem to prove that the alternating sum of dimensions, represented as ∑(-1)^{i}d_{i}, equals zero for a sequence of matrices where the image of one matrix equals the kernel of the next. Participants emphasize the importance of applying the rank-nullity theorem correctly to derive the necessary equations. The key approach involves setting up equations based on the relationships between the images and dimensions of the matrices. By manipulating these equations, the goal is to show that the alternating sum simplifies to zero. The conversation highlights the need for careful calculation and substitution to arrive at the final result.
bmanbs2
Messages
22
Reaction score
0

Homework Statement



Consider integer sequence n_{1},...,n_{r} and matrices A_{1},...,A_{n-1}. Assume im\left(A_{i}\right) = ker\left(A_{i+1}\right)

Using the rank-nullity theorem, show that \sum^{n}_{i=1}\left(-1\right)^{i}d_{i} = 0


Homework Equations



The rank-nullity theorem states that if v and w are vector spaces and A is the linear map A: v -> w, then
dim\left(im\left(A\right)\right) + dim\left(ker\left(A\right)\right) = dim\left(v\right)

The Attempt at a Solution



I know that the relation im\left(A_{i}\right) = ker\left(A_{i+1}\right) means that \sum^{n}_{i even}d_{i} = \sum^{n}_{i odd}d_{i}, but I don't know how to get there.
 
Physics news on Phys.org
First, apply rank-nullity on A1,...,An. You'll get n equations. Adding up those equations should get you the answer...
 
Thanks for the reply, but I want to make sure I'm setting up the equations properly.

So far I have that
im\left(A_{i-1}\right) + im\left(A_{i}\right) = d_{i}
im\left(A_{i}\right) + im\left(A_{i+1}\right) = d_{i+1}

Subtracting the system of equations gives
im\left(A_{i-1}\right) - im\left(A_{i+1}\right) = d_{i} - d_{i+1}

That may then be continued for 2 \leq i \leq n-1, but I'm not sure how to make it all equal zero.
 
OK, I was wrong about adding up the equations. But what you have looks good:

im(A_{i-1})+im(A_i)=d_i.

Now, you have to calculate

-d_1+d_2-d_3+d_4-...

Just substitute every di in the above addition by im(A_{i-1})+im(A_i), and make a little calculation.
 
Question: A clock's minute hand has length 4 and its hour hand has length 3. What is the distance between the tips at the moment when it is increasing most rapidly?(Putnam Exam Question) Answer: Making assumption that both the hands moves at constant angular velocities, the answer is ## \sqrt{7} .## But don't you think this assumption is somewhat doubtful and wrong?

Similar threads

Replies
15
Views
2K
  • · Replies 8 ·
Replies
8
Views
2K
Replies
2
Views
1K
  • · Replies 4 ·
Replies
4
Views
2K
  • · Replies 8 ·
Replies
8
Views
2K
  • · Replies 9 ·
Replies
9
Views
3K
  • · Replies 14 ·
Replies
14
Views
2K
  • · Replies 4 ·
Replies
4
Views
1K
  • · Replies 13 ·
Replies
13
Views
2K
  • · Replies 4 ·
Replies
4
Views
2K